Evidence

अब Quizwiz के साथ अपने होमवर्क और परीक्षाओं को एस करें!

In an automobile collision case, the defendant's attorney called the defendant to the stand and asked, "Was the traffic light red, amber, or green when you entered the intersection?" The defendant replied, "It was green." Next, the defendant's attorney asked, "What did you tell the first police officer who arrived on the scene about the condition of the traffic light when you entered the intersection?" Before the defendant could reply with "I told him it was green," the plaintiff's attorney objected. The objection should be: (A) Sustained, because the statement is hearsay not within any recognized exception to the hearsay rule. (B) Sustained, because the testimony is an irrelevant prior consistent statement. (C) Overruled, because the statement is made from personal knowledge and, therefore, is non-hearsay. (D) Overruled, because the defendant is in court and is subject to cross-examination by the plaintiff's attorney.

(A) Sustained, because the statement is hearsay not within any recognized exception to the hearsay rule.

In a personal injury case involving a two-car collision, the plaintiff wishes to introduce a sworn deposition taken from a witness who died two weeks before the case came to trial. In the deposition, the witness stated that she saw the defendant run a red light at the time of the collision with the plaintiff's car. Both the plaintiff's and the defendant's attorneys were present at the deposition. The defendant objects in the appropriate manner to the introduction of the witness's statement. How should the court rule on the admissibility of the deposition? (A) Admissible, because the defendant had an opportunity to cross-examine the witness at the time the deposition was taken. (B) Admissible. as a dying declaration. (C) Inadmissible, because the statement was not made while the witness was testifying in court. (D) Inadmissible, because the defendant has no opportunity to cross-examine the witness at trial.

(A) Admissible, because the defendant had an opportunity to cross-examine the witness at the time the deposition was taken.

In a prosecution of Dale for murdering Vera, Dale testified that the killing had occurred in self-defense when Vera tried to shoot him. In rebuttal, the prosecution seeks to call Walter, Vera's father, to testify that the day before the killing, Vera told Walter that she loved Dale so much she could never hurt him. Walter's testimony is (A) admissible within the hearsay exception for statements of the declarant's then existing state of mind. (B) admissible, because Vera is unavailable as a witness. (C) inadmissible as hearsay not within any exception. (D) inadmissible, because Vera's character is not an issue.

(A) admissible within the hearsay exception for statements of the declarant's then existing state of mind.

A defendant is on trial for attempted fraud. The state charges that the defendant switched a price tag from a cloth coat to a more expensive fur-trimmed coat and then presented the latter for purchase at the cash register. The defendant has testified in her own behalf that someone else must have switched the tag. On cross-examination, the prosecutor asks whether the defendant was convicted on two prior occasions of misdemeanor fraud of a retailer by the same means of switching the price tag on a fur-trimmed coat. Is the question about the prior convictions proper either to impeach the defendant or to prove that the defendant committed the crime? (A) It is not proper for either purpose. (B) It is proper for both purposes. (C) It is proper to impeach the defendant, but not to prove that the defendant committed the crime. (D) It is proper to prove that the defendant committed the crime, but not to impeach the defendant. -In a common law jurisdiction, what crime did the defendant commit (or at least attempt to commit)?

(B) It is proper for both purposes. -False pretenses

A defendant was charged with the crime of defrauding the federal agency where he worked as an accountant. At trial, the court allowed the defendant to call his supervisor at the large corporation where he had previously worked, who testified about the defendant's good reputation in the community for honesty. Over objection, the defendant then sought to elicit testimony from his former supervisor that on several occasions the corporation had, without incident, entrusted him with large sums of money. Should the testimony be admitted? (A) No, because the testimony is extrinsic evidence on a collateral matter. (B) No, because good character cannot be proved by specific instances of conduct unless character is an essential element of the charge or defense. (C) Yes, because it is evidence of a pertinent character trait offered by an accused. (D) Yes, because it is relevant to whether the defendant was likely to have taken money as charged in this case.

(B) No, because good character cannot be proved by specific instances of conduct unless character is an essential element of the charge or defense.

A defendant is charged with robbing a bank. The prosecutor has supplied the court with information from accurate sources establishing that the bank is a federally insured institution and that this fact is not subject to reasonable dispute. The prosecutor asks the court to take judicial notice of this fact. The defendant objects. How should the court proceed? (A) The court must take judicial notice and instruct the jury that it is required to accept the judicially noticed fact as conclusive. (B) The court must take judicial notice and instruct the jury that it may, but is not required to, accept the judicially noticed fact as conclusive. (C) The court may refuse to take judicial notice, because judicial notice may not be taken of essential facts in a criminal case. (D) The court must refuse to take judicial notice, because whether a bank is federally insured would not be generally known within the court's jurisdiction.

(B) The court must take judicial notice and instruct the jury that it may, but is not required to, accept the judicially noticed fact as conclusive.

A defendant was charged with possession of marijuana with intent to distribute. On direct examination, the defendant testified that he worked with disadvantaged children as a drug counselor, that he hated drugs, that he would "never possess or distribute drugs," and that he had never used drugs and would not touch them. The government offered as a rebuttal witness a police officer who would testify that, three years earlier, he saw the defendant buy cocaine from a street dealer. The defendant objected. Is the testimony of the police officer about the prior drug transaction admissible to impeach the defendant? (A) No, because the bad act of buying drugs is not sufficiently probative of a witness's character for truthfulness. (B) No, because it is contradiction on a collateral matter. (C) Yes, because it is proper contradiction. (D) Yes, because the bad act shows a disregard for the law and makes it less likely that the defendant would respect the oath of truthfulness.

(C) Yes, because it is proper contradiction.

Park sued Officer Dinet for false arrest. Dinet's defense was that, based on a description he heard over the police radio, he reasonably believed Park was an armed robber. Police radio dispatcher Brigg, reading from a note, broadcasted the description of an armed robber on which Dinet claims to have relied. The defendant offers the following items of evidence: I. Dinet's testimony relating the description he heard. II. Brigg's testimony relating the description he read over the radio. III. The note containing the description Brigg testifies he read over the radio. Which of the following are admissible on the issue of what description Dinet heard? (A) I and II only. (B) I and III only. (C) II and III only. (D) I, II, and III.

(D) I, II, and III.

Plaintiff sued Defendant for breach of a commercial contract in which Defendant had agreed to sell Plaintiff all of Plaintiff's requirements for widgets. Plaintiff called Expert Witness to testify as to damages. Defendant seeks to show that Expert Witness had provided false testimony as a witness in his own divorce proceedings. This evidence should be A. Admitted only if elicited from Expert Witness on cross-examination. B. Admitted only if the false testimony is established by clear and convincing extrinsic evidence. C. Excluded, because it is impeachment on a collateral issue. D. Excluded, because it is improper character evidence.

A. Admitted only if elicited from Expert Witness on cross-examination.

Dalton is on trial for burglary. During cross-examination of Dalton, the prosecutor wants to inquire about Dalton's earlier conviction for falsifying a credit application. Which of the following facts concerning the conviction would be the best reason for the trial court's refusing to allow such examination? A. Dalton was released from prison 12 years ago. B. Dalton was put on probation rather than imprisoned. C. It was for a misdemeanor rather than a felony. D. It is on appeal.

A. Dalton was released from prison 12 years ago.

Defendant is charged with arson of a church. His defense is that another individual, Frank, set fire to the church, based solely on the testimony of a Witness. The Witness, who works with Frank, will testify that Frank told him that he set fire to the church. The prosecutor's objection to the Witness's testimony should be: A. Sustained, if there is no corroboration of Frank's statement. B. Overruled, if defendant can show that Frank is unavailable as a witness. C. Overruled, because it is a declaration against interest. D. Overruled, because Frank's statement to the Witness was not made to a police officer.

A. Sustained, if there is no corroboration of Frank's statement.

Dirk is on trial for the brutal murder of Villas. Dirk's first witness, Wesley, testified that in her opinion Dirk is a peaceful and nonviolent person. The prosecution does not cross-examine Wesley, who is then excused from further attendance. Which one of the following is INADMISSIBLE during the prosecution's rebuttal? A. Testimony by Wesley's former employer that Wesley submitted a series of false expense vouchers two years ago. B. Testimony by a police officer that Dirk has a long-standing reputation in the community as having a violent temper. C. Testimony by a neighbor that Wesley has a long-standing reputation in the community as an untruthful person. D. Testimony by Dirk's former cell mate that he overheard Wesley offer to provide favorable testimony if Dirk would pay her $5,000.

A. Testimony by Wesley's former employer that Wesley submitted a series of false expense vouchers two years ago.

In litigation on a federal claim, Plaintiff had the burden of proving that Defendant received a notice. Plaintiff relied on the presumption of receipt by offering evidence that the notice was addressed to Defendant, properly stamped, and mailed. Defendant, on the other hand, testified that she never received the notice. Which of the following is correct? A. The jury must find that the notice was received. B. The jury may find that the notice was received. C. The burden shifts to Defendant to persuade the jury of non-receipt. D. The jury must find that the notice was not received, because the presumption has been rebutted and there is uncontradicted evidence of non-receipt.

B. The jury may find that the notice was received.

Paul sued Donna for breach of contract. Paul's position was that Joan, whom he understood to be Donna's agent, said: "On behalf of Donna, I accept your offer." Donna asserted that Joan had no actual or apparent authority to accept the offer on Donna's behalf. Paul's testimony concerning Joan's statement is A. admissible, provided the court first finds by a preponderance of the evidence that Joan had actual or apparent authority to act for Donna. B. admissible upon or subject to introduction of evidence sufficient to support a finding by the jury that Joan had actual or apparent authority to act for Donna. C. inadmissible, if Joan does not testify and her absence is not excused. D. inadmissible, because it is hearsay not within an exception.

B. admissible upon or subject to introduction of evidence sufficient to support a finding by the jury that Joan had actual or apparent authority to act for Donna.

In a civil action for personal injury, Payne alleges that he was beaten up by Dabney during an altercation in a crowded bar. Dabney's defense is that he was not the person who hit Payne. To corroborate his testimony about the cause of his injuries, Payne seeks to introduce, through the hospital records custodian, a notation in a regular medical record made by an emergency room doctor at the hospital where Payne was treated for his injuries. The notation is: "Patient says he was attacked by Dabney." The notation is A. inadmissible, unless the doctor who made the record is present at trial and available for cross-examination. B. inadmissible as hearsay not within any exception. C. admissible as hearsay within the exception for records of regularly conducted activity. D. admissible as a statement made for the purpose of medical diagnosis or treatment.

B. inadmissible as hearsay not within any exception.

At the trial of a civil action between plaintiff and defendant following an intersection collision between their cars, plaintiff calls witness, who was the only other witness to the accident. Just after plaintiff's attorney asks witness a question, defendant's attorney says, "I object, your honor. That question is clearly improper." The court overruled the objection. Defendant loses the case, and appeals the judgment on the basis of the court's ruling on the objection. Which of the following statements is most accurate? A. Because defendant objected at the appropriate time, the appellate court will reverse if any error in the court's ruling affected a substantial right of defendant. B. If the record clearly reflects the nature of the error, the appellate court will reverse because witness's testimony was crucial to plaintiff's case. C. Unless the record clearly reflects both the nature of the error and that it affected a substantial right of defendant, the appellate court will not reverse. D. Because defendant did not state the ground for the objection, the appellate court will not reverse.

C. Unless the record clearly reflects both the nature of the error and that it affected a substantial right of defendant, the appellate court will not reverse.

At Devlin's trial for burglary, Jack supported Devlin's alibi that they were fishing together at the time of the crime. On cross-examination, Jack was asked whether his statement on a credit card application that he had worked for his present employer for the last five years was false. Jack denied that the statement was false. The prosecutor then calls Wilcox, the manager of the company for which Jack works, to testify that although Jack had been first employed five years earlier and is now employed by the company, there had been a three-year period during which he had not been so employed. The testimony of Wilcox is A. admissible, in the judge's discretion, because Jack's credibility is a fact of major consequence to the case. B. admissible, as a matter of right, because Jack "opened the door" by his denial on cross-examination. C. inadmissible, because whether Jack lied in his application is a matter that cannot be proved by extrinsic evidence. D. inadmissible, because the misstatement by Jack could have been caused by a misunderstanding of the application form.

C. inadmissible, because whether Jack lied in his application is a matter that cannot be proved by extrinsic evidence.

Plaintiff sued Defendant for personal injuries suffered in a train-automobile collision. Plaintiff called an eyewitness, who testified that the train was going 20 miles per hour. Defendant then offers the testimony of an experienced police accident investigator that, based on his training and experience and on his examination of the physical evidence, it is his opinion that the train was going between 5 and 10 miles per hour. Testimony by the investigator is A. improper, because there cannot be both lay and expert opinion on the same issue. B. improper, because the investigator is unable to establish the speed with a sufficient degree of scientific certainty. C. proper, because a police accident investigator has sufficient expertise to express an opinion on speed. D. proper, because Plaintiff first introduced opinion evidence as to speed.

C. proper, because a police accident investigator has sufficient expertise to express an opinion on speed.

Defendant is on trial for participating in a drug sale. The prosecution calls Witness, an undercover officer, to testify that, when Seller sold the drugs to Witness, Seller introduced Defendant to Witness as "my partner in this" and Defendant shook hands with Witness but said nothing. Witness's testimony is A. inadmissible, because there is no evidence that Seller was authorized to speak for Defendant. B. inadmissible, because the statement of Seller is hearsay not within any exception. C. admissible as a statement against Defendant's penal interest. D. admissible as Defendant's adoption of Seller's statement.

D. admissible as Defendant's adoption of Seller's statement.

Pedestrian died from injuries caused when Driver's car struck him. Executor, Pedestrian's executor, sued Driver for wrongful death. At trial, Executor calls Nurse to testify that two days after the accident, Pedestrian said to Nurse, "The car that hit me ran the red light." Fifteen minutes thereafter, Pedestrian died. As a foundation for introducing evidence of Pedestrian's statement, Executor offers to the court Doctor's affidavit that Doctor was the intern on duty the day of the Pedestrian's death and that several times that day Pedestrian had said that he knew he was about to die. Is the affidavit properly considered by the court in ruling on the admissibility of Pedestrian's statement? A. No, because it is hearsay not within any exception. B. No, because it is irrelevant since dying declarations cannot be used except in prosecutions for homicide. C. Yes, because, though hearsay, it is a statement of then-existing mental condition. D. Yes, because the judge may consider hearsay in ruling on preliminary questions.

D. Yes, because the judge may consider hearsay in ruling on preliminary questions.

D was tried for the July 1 murder of V. D did not testify at trial. D called Y to testify to alibi. On cross-examination of Y, the prosecutor asked, "Isn't it a fact that you once cheated on a law school exam?" The question is a. proper because it goes to bias. b. proper to show that Y is untruthful. c. improper because the question goes beyond the scope of direct examination. d. improper because the evidence being sought is irrelevant.

b. proper to show that Y is untruthful.

D was tried for the July 1 murder of V. D did not testify at trial. D called X to testify that on July 2 D told him (X) that he (D) had just returned from a three-day visit to relatives in a distant state. The testimony is a. admissible because it is a declaration of present mental state. b. admissible because it is not hearsay. c. inadmissible because it is irrelevant. d. inadmissible because it is hearsay not within any exception.

d. inadmissible because it is hearsay not within any exception.

D was tried for the July 1 murder of V. D did not testify at trial. In his case-in-chief, D called as his first witness W to testify to D's reputation in his community as an "honest man." The testimony is a. admissible as tending to prove D is believable. b. admissible as tending to prove D is innocent. c. admissible as proper opinion testimony. d. inadmissible.

d. inadmissible.

D was tried for the July 1 murder of V. D did not testify at trial. D called Z to testify to alibi. On cross-examination of Z, the prosecutor asked, "Isn't it true that you were convicted of perjury last year?" This question is a. improper because it goes beyond the scope of direct examination. b. improper, unless its probative value outweighs the prejudice to D. c. proper, but only if the maximum punishment for perjury was more than one year imprisonment. d. proper.

d. proper.

A sportscaster on a local television show interviewed the parent of a child on a high school football team. The interviewee told the sportscaster that the head football coach "openly condones the use of steroids by team members." The coach, who had always conducted a strong anti-drug program for his football players, watched and recorded the show daily. He was outraged when he saw the live broadcast, and filed suit for defamation against the interviewee, the sportscaster, and the television station. At the trial of the suit, the coach wishes to testify as to what the interviewee said on the television show. The defense objects. Should such testimony be admitted? (A) Yes, because the coach saw the live television broadcast. (B) Yes, because the matter goes to the ultimate issue of the case and is thus highly relevant. (C) No, because a videotape of the broadcast is available. (D) No, because such testimony would be hearsay, not within any recognized exception to the hearsay rule.

(A) Yes, because the coach saw the live television broadcast.

Plaintiff sued Defendant for illegal discrimination, claiming that Defendant fired him because of his race. At trial, Plaintiff called Witness, expecting him to testify that Defendant had admitted the racial motivation. Instead, Witness testified that Defendant said that he had fired Plaintiff because of his frequent absenteeism. While Witness is still on the stand, Plaintiff offers a properly authenticated secret tape recording he had made at a meeting with Witness in which Witness related Defendant's admissions of racial motivation. The tape recording is A. Admissible as evidence of Defendant's racial motivation and to impeach Witness's testimony. B. Admissible only to impeach Witness's testimony. C. Inadmissible, because it is hearsay not within any exception. D. Inadmissible, because a secret recording is an invasion of Witness's right of privacy under the U.S. Constitution.

B. Admissible only to impeach Witness's testimony.

Daggett was prosecuted for murder of Vales, whose body was found one morning in the street near Daggett's house. The state calls Witt, a neighbor, to testify that during the night before the body was found he heard Daggett's wife scream, "You killed him! You killed him!" Witt's testimony is A. admissible as a report of a statement of belief. B. admissible as a report of an excited utterance. C. inadmissible, because it reports a privileged spousal communication. D. inadmissible on spousal immunity grounds, but only if the wife objects.

B. admissible as a report of an excited utterance.

On trial for murdering her husband, Defendant testified she acted in self-defense. Defendant calls Expert, a psychologist, to testify that under hypnosis Defendant had described the killing, and that in Expert's opinion Defendant had been in fear for her life at the time of the killing. Is Experts' testimony admissible? A. Yes, because Expert was able to ascertain that Defendant was speaking truthfully. B. Yes, because it reports a prior consistent statement by a witness (Defendant) subject to examination concerning it. C. No, because reliance on information tainted by hypnosis is unconstitutional. D. No, because it expresses an opinion concerning Defendant's mental state at the time of the killing.

D. No, because it expresses an opinion concerning Defendant's mental state at the time of the killing.

W was driving west on I-40 when she noticed D pass her at about 80 miles per hour. About a minute later she passed the scene of an accident involving D and another vehicle (driven by P). W was called to testify in an action by P that she had observed D approximately a mile from the accident traveling at an excessive speed. D's speed at the time of the accident is a material issue in the trial. W's testimony should be a. admitted, because it tends to prove that D was speeding at the time of the accident. b. admitted, but only if W is qualified as an expert at judging vehicular speed. c. excluded, unless it is proved that D has a habit of speeding every time he travels on the highway. d. excluded, because it is character evidence offered in a civil trial.

a. admitted, because it tends to prove that D was speeding at the time of the accident.

P was driving along Elm Street at 35 m.p.h. (the posted speed limit). As he approached the intersection with 13th Street, a car driven by S entered from 13th Street at 50 m.p.h. The two cars crashed. P, who was seriously injured in the accident, brought a personal injury action against S. At trial, P called Nurse to the stand. Nurse testified that on the date of the accident she was on duty in the hospital emergency room when P was admitted. She further testified that she saw Dr. Dole treat P's broken hip. Immediately before taking the witness stand, Nurse refreshed her recollection by studying the hospital records that had not yet been admitted into evidence. Upon objection by S's attorney, Nurse's testimony is a. admissible as present recollection recorded. b. admissible as evidence of the nature of P's injuries. c. inadmissible, because it is based on hearsay not within an exception. d. inadmissible, because it is not the best evidence of P's injuries.

b. admissible as evidence of the nature of P's injuries.

Peters sued Davis for $100,000 for injuries received in a traffic accident. Davis charged Peters with contributory negligence and alleged that Peters failed to have his lights on at a time when it was dark enough to require them. Davis offers to have Bystander testify that he was talking to Witness when, just before Bystander heard the crash, Bystander heard Witness, now deceased, exclaim, "That car doesn't have any lights on." Bystander's testimony is (A) admissible as a statement of present sense impression. (B) admissible, because Witness is not available to testify. (C) inadmissible as hearsay, not within any exception. (D) inadmissible, because of the Dead Man's Statute.

(A) admissible as a statement of present sense impression.

Penn sued Duke's Bar for injuries suffered in an automobile accident caused by Chase, who had been a patron of Duke's Bar. Penn claimed that Chase was permitted to drink too much liquor at Duke's Bar before the accident. Penn offered evidence that, after the accident, the owner of Duke's Bar visited him at the hospital and, offering to pay all of Penn's medical expenses, said, "That's the least I can do after letting Chase leave the bar so drunk last night." The statement that Chase was drunk when he left the bar on the night of the accident is (A) admissible as an admission by the owner of Duke's Bar that Chase was drunk when he left the bar. (B) admissible as a factual admission made in connection with an offer of compromise. (C) inadmissible as hearsay, not within any exception. (D) inadmissible as a statement made in connection with an offer to pay medical expenses.

(A) admissible as an admission by the owner of Duke's Bar that Chase was drunk when he left the bar.

Porter sued Data Exchange, a computer dealer engaged in buying and selling used computers, alleging that he was not given credit for a CD-ROM drive on the computer that he had sent back for resale. Warden, the bookkeeper, testified that it was company practice when a boxed computer was returned to have one clerk open the box and identify the type of computer and its components and have another clerk record the information in the inventory ledger. Data Exchange seeks to enter into evidence the original ledger entry, which Warden authenticated, showing that a CD-ROM drive was not checked off on the components list for Porter's computer. Porter objects to the admission of the ledger. The ledger is: (A) Admissible, because it is a record of a transaction for which Warden does not have any present recollection. (B) Admissible, because it was regular company practice to record receipt of the components in the inventory ledger. (C) Inadmissible as hearsay within hearsay, because even if a hearsay exception permits introducing the record itself rather than requiring testimony by the employee who made it, that employee was just recording hearsay because he had no personal knowledge of what he was recording. (D) Inadmissible hearsay, because absence of the notation implies a statement that no CD-ROM was received, and the evidence is being offered as proof of that assertion.

(B) Admissible, because it was regular company practice to record receipt of the components in the inventory ledger.

A plaintiff has brought a civil suit against a defendant for injuries arising out of a fistfight between them. The day after the fight, a police officer talked to the plaintiff, the defendant, and an eyewitness, and made an official police report. At trial, the plaintiff seeks to introduce from the properly authenticated police report a statement attributed to the eyewitness, who is unavailable to testify at trial, that "[the defendant] started the fight." Should the court admit the statement from the report? (A) No, unless the entire report is introduced. (B) No, because it is hearsay not within any exception. (C) Yes, because it was based on the eyewitness's firsthand knowledge. (D) Yes, because it is an excerpt from a public record offered in a civil case. -If the plaintiff seeks to introduce from the properly authenticated police report a statement attributed to the defendant, should the court admit that statement from the report?

(B) No, because it is hearsay not within any exception. -Yes, the report itself is admissible as a business or public record, but the hearsay within it is admissible only if it satisfies a separate hearsay exception. Here, the defendant's statement is admissible as a statement of a party-opponent.

A defendant was prosecuted for mail fraud. At trial, the defendant moved to have all witnesses excluded from the courtroom, and the court granted the motion. The government named the investigating FBI agent as its designated representative. Upon learning that the agent would be giving testimony during the trial, the defendant moved that the agent also be excluded from the courtroom. Should the defendant's motion be granted? (A) No, provided that the government can show that the agent's presence is essential to the presentation of its case. (B) No, because the government has a right to have its designated representative remain in the courtroom throughout the trial. (C) Yes, because the agent's testimony might be influenced by the testimony of other witnesses. (D) Yes, because the defendant has a right to exclude all persons who may be called as government witnesses.

(B) No, because the government has a right to have its designated representative remain in the courtroom throughout the trial.

At a civil trial for slander, the plaintiff showed that the defendant had called the plaintiff a thief. In defense, the defendant called a witness to testify, "I have been the plaintiff's neighbor for many years, and people in our community generally have said that he is a thief." Is the testimony concerning the plaintiff's reputation in the community admissible? (A) No, because character is an essential element of the defense, and proof must be made by specific instances of conduct. (B) Yes, to prove that the plaintiff is a thief and to reduce or refute the damages claimed. (C) Yes, to prove that the plaintiff is a thief, but not on the issue of damages. (D) Yes, to reduce or refute the damages claimed, but not to prove that the plaintiff is a thief.

(B) Yes, to prove that the plaintiff is a thief and to reduce or refute the damages claimed.

In litigation over the estate of Baggs, who died intestate, Payton, who is 18 years old, claimed to be Baggs's niece and entitled, therefore, to a share of his large estate. In support of her claim, Payton offered in evidence a Bible, properly identified as having belonged to Baggs's family, in the front of which was a list of family births, marriages, and deaths. The list recorded Payton's birth to Baggs's oldest sister. To prove that Payton is Baggs's niece, the Bible listing is (A) admissible as an ancient document. (B) admissible as a family record. (C) inadmissible, because it is hearsay, not within any exception. (D) inadmissible, because there was no showing of first-hand knowledge by the one who wrote it.

(B) admissible as a family record.

P sued D for a libelous letter received by Investigator. The authenticity of the letter is disputed. To establish authenticity, P's attorney asks Investigator to testify that, a week before receiving the libelous letter, he had written to D inquiring about P. The testimony is (A) admissible provided that this inquiry was made in the regular course of Investigator's business. (B) admissible without production of the inquiry letter or the showing of its unavailability. (C) inadmissible unless P's attorney has given D notice of Investigator's intended testimony. (D) inadmissible unless the inquiry letter itself is shown to be unavailable.

(B) admissible without production of the inquiry letter or the showing of its unavailability.

Deeb was charged with stealing furs from a van. At trial, Wallace testified she saw Deeb take the furs. The jurisdiction in which Deeb is being tried does not allow in evidence lie detector results. On cross-examination by Deeb's attorney, Wallace was asked, "The light was too dim to identify Deeb, wasn't it?" She responded, "I'm sure enough that it was Deeb that I passed a lie detector test administered by the police." Deeb's attorney immediately objects and moves to strike. The trial court should (A) grant the motion, because the question was leading. (B) grant the motion, because the probative value of the unresponsive testimony is substantially outweighed by the danger of unfair prejudice. (C) deny the motion, because it is proper rehabilitation of an impeached witness. (D) deny the motion, because Deeb's attorney "opened the door" by asking the question.

(B) grant the motion, because the probative value of the unresponsive testimony is substantially outweighed by the danger of unfair prejudice.

A plaintiff suffered injuries when her car was struck by the defendant's car. The police arrived on the scene and required the defendant to take a breathalyzer test. The defendant was cited for driving while intoxicated, tried in traffic court, and duly convicted. He received the maximum sentence for driving while intoxicated, which is 90 days' imprisonment in the county correctional facility. The plaintiff brought a civil action against the defendant, seeking compensation for her personal injuries. At the trial of the plaintiff's suit, the plaintiff's attorney offers a properly authenticated photocopy of the court judgment showing that the defendant was convicted of driving while intoxicated. The evidence is: (A) Admissible as a public record. (B) Admissible as a final judgment offered to prove a fact essential to a point in controversy. (C) Inadmissible, because the crime was punishable by imprisonment of a maximum of 90 days. (D) Inadmissible, because it is not the best evidence of the defendant's conviction. -If defendant were convicted of "Felony DUI," which answer would be correct?

(C) Inadmissible, because the crime was punishable by imprisonment of a maximum of 90 days. -(B)

The plaintiff is suing the defendant in federal court for personal injuries arising out of an assault and battery. The defendant has confided in his psychiatrist certain matters that the plaintiff believes would be helpful to her case. Assume the information is privileged, so the psychiatrist cannot be called at trial, but it is relevant and would lead to admissible evidence. If the defendant objects, will the federal court order discovery of what plaintiff told his psychiatrist? (A) Yes, because the information from the psychiatrist will lead to other discoverable evidence. (B) Yes, if the judge orders that the records from the psychiatrist be sealed. (C) No, because the communication with the psychiatrist is privileged. (D) No, if the psychiatrist waives the privilege.

(C) No, because the communication with the psychiatrist is privileged.

A man sued a railroad for personal injuries suffered when his car was struck by a train at an unguarded crossing. A major issue is whether the train sounded its whistle before arriving at the crossing. The railroad has offered the testimony of a resident who has lived near the crossing for 15 years. Although she was not present on the occasion in question, she will testify that, whenever she is home, the train always sounds its whistle before arriving at the crossing. Is the resident's testimony admissible? (A) No, due to the resident's lack of personal knowledge regarding the incident in question. (B) No, because habit evidence is limited to the conduct of persons, not businesses. (C) Yes, as evidence of a routine practice. (D) Yes, as a summary of her present sense impressions.

(C) Yes, as evidence of a routine practice.

A motorist who failed to stop at a stop sign was struck by a car being taken for a test drive by a mechanic who had repaired the car's brakes. The motorist sued the repair shop that employed the mechanic to recover for his injuries. At trial, he called a bystander to testify that when the mechanic saw that the motorist was injured, the mechanic ran over and told him, "I'm really sorry. I guess I didn't fix the brakes as well as I thought." The mechanic is available to testify at the trial. The repair shop's objection to the bystander's testimony should be: (A) Sustained, because the mechanic's statement is inadmissible against the repair shop. (B) Sustained, because the motorist did not stop at the stop sign. (C) Overruled, because it is a declaration against interest. (D) Overruled, because it is a statement attributable to a party-opponent.

(D) Overruled, because it is a statement attributable to a party-opponent.

In a suit based on a will, the distribution of $1 million depends upon whether the wife survived her husband when both died in the crash of a small airplane. An applicable statute provides that, for purposes of distributing an estate after a common disaster, there is a rebuttable presumption that neither spouse survived the other. A witness has been called to testify that as she approached the plane she heard what she thought was a woman's voice saying, "I'm dying," although by the time the husband and wife were removed from the wreckage they were both dead. Is the witness's testimony admissible? (A) No, because the matter is governed by the presumption that neither spouse survived the other. (B) No, because the witness's testimony is too speculative to support a finding. (C) Yes, because the hearsay rule does not apply to statements by decedents in actions to determine rights under a will. (D) Yes, because it is relevant and not otherwise prohibited. -Why is the dying woman's statement not hearsay?

(D) Yes, because it is relevant and not otherwise prohibited. -It is not being offered for its truth but only to prove that she could speak and therefore was alive.

Plaintiff sues defendant for negligence arising out of an automobile collision. As part of her case-in-chief, plaintiff calls defendant to the stand. During the direct examination of defendant, plaintiff's attorney asks, "Isn't it true that at the time of the accident you were going 65 mph?" Which of the following statements about this question is accurate? A. The question is leading but proper. B. The question is an improper leading question. C. The question is not leading and is proper. D. The question is not leading but assumes a fact not in evidence. -On cross-examination of defendant, may defendant's attorney ask leading questions?

A. The question is leading but proper. -Answer: No.

A plaintiff sued a defendant, alleging that she was seriously injured when the defendant ran a red light and struck her while she was walking in a crosswalk. During the defendant's case, a witness testified that the plaintiff had told him that she was "barely touched" by the defendant's car. On cross-examination, should the court allow the plaintiff to elicit from the witness the fact that he is an adjuster for the defendant's insurance company? (A) No, because testimony about liability insurance is barred by the rules of evidence. (B) No, because the reference to insurance raises a collateral issue. (C) Yes, for both substantive and impeachment purposes. (D) Yes, for impeachment purposes only.

(D) Yes, for impeachment purposes only.

Decker, charged with armed robbery of a store, denied that he was the person who had robbed the store. In presenting the state's case, the prosecutor seeks to introduce evidence that Decker had robbed two other stores in the past year. This evidence is (A) admissible, to prove a pertinent trait of Decker's character and Decker's action in conformity therewith. (B) admissible, to prove Decker's intent and identity. (C) inadmissible, because the prosecutor may rely only on reputation or opinion evidence as this point in the trial to prove the defendant's character. (D) inadmissible, because its probative value is substantially outweighed by the danger of unfair prejudice.

(D) inadmissible, because its probative value is substantially outweighed by the danger of unfair prejudice.

In a wrongful death action for the death of his wife in an automobile accident, the plaintiff alleged that the accident was caused by a mud-flap assembly that fell off the defendant's truck. The plaintiff wishes to introduce the testimony of a witness, another truck driver who was on the same highway at the time, who heard someone tell the defendant over CB radio that he had noticed at the truck stop that the defendant's mud-flap assembly on his truck was loose. The witness does not know the identity of the person who gave the warning. If the defendant objects to the admission of this testimony, the court should rule that it is: A. Admissible to prove that the defendant was notified that the mud-flap assembly was loose. B. Admissible both to prove that the defendant was notified that the mud-flap assembly was loose and as substantive evidence that it was loose. C. Inadmissible, because the witness cannot identify who made the statement. D. Inadmissible, because it is hearsay not within any recognized exception.

A. Admissible to prove that the defendant was notified that the mud-flap assembly was loose.

A defendant, a nurse at a nursing home, is charged with murdering a resident at the home by adding an allegedly lethal substance to the resident's food. At trial, to prove that the substance added to the resident's food could result in death, the prosecutor, without first calling any witnesses, offers to read into evidence several pages from a standard medical treatise that support the prosecution's claim that the substance the defendant added to the food is lethal. Is the evidence offered admissible? A. No, because the treatise excerpts were not offered during the examination of a qualified expert. B. No, because the treatise itself must be introduced as an exhibit. C. Yes, under the learned treatise exception to the hearsay rule. D. Yes, because the lethal nature of the substance is relevant to the defendant's state of mind and intent.

A. No, because the treatise excerpts were not offered during the examination of a qualified expert.

In a negligence action, plaintiff sues defendant, a heating and cooling company, for injuries she suffered when defendant allegedly failed to fix plaintiff's heater during a sub-zero cold snap. Plaintiff, who is elderly and housebound, alleges that she phoned defendant as soon as the heater failed, and that defendant promised to respond "within two hours," the plaintiff waited in vain for the entire night, and the defendant never appeared. While waiting for defendant, plaintiff alleges that she suffered cold-related injuries to her fingers and toes. Plaintiff testified that she placed her call to the number for defendant listed in the phone book, and that a voice answered, "heating service." Defendant denies ever receiving the call, and moves to strike plaintiff's testimony from the record. How should the court rule? A. The court should deny defendant's motion because there is sufficient evidence to support a finding that the call was actually placed to defendant. B. The court should deny the motion if it finds that the call was not placed to defendant. C. The court should grant the motion because the party answering the phone did not identify itself as defendant's heating and cooling company. D. The court should deny the motion because plaintiff has already testified, and it is too late to keep the evidence from the jury.

A. The court should deny defendant's motion because there is sufficient evidence to support a finding that the call was actually placed to defendant.

In a fraud action by plaintiff against defendant, a car dealership, plaintiff claims that the car defendant sold to her got only 11 miles per gallon, even though defendant's salesperson told plaintiff that it would get "in the mid-20s." Defendant denies its salesperson made such representation. To prove the representation was made, plaintiff wishes to offer the testimony of five other customers that the salesperson made the same representation to them about the same model of car. Defendant does not deny that the other representations were made, but objects to admission of the evidence. How should the court rule? A. The court should overrule the objection because the evidence is admissible pattern evidence to show that the representation was made. B. The court should overrule the objection because the evidence is admissible habit evidence. C. The court should sustain the objection because the evidence is inadmissible character evidence. D. The court should sustain the objection because the evidence, though relevant non-character evidence, is inadmissible because this is a civil action.

A. The court should overrule the objection because the evidence is admissible pattern evidence to show that the representation was made.

Plaintiff sues defendant for breach of contract. Plaintiff claims the defendant breached the written contract by failing to deliver 200 crates of apples by the specified date. At trial, defendant (without offering the written contract into evidence) wishes to testify that the contract called for delivery of only 100 crates. Plaintiff makes a best evidence rule objection. How should the court rule? A. The court should sustain the objection. B. The court should overrule the objection because a contract is an agreement brought about by the meeting of two minds, not a writing. The writing is only a memorandum of the agreement, not the contract itself. C. The court should overrule the objection because defendant is not testifying about the contents of the writing, but about her understanding of the agreement. Thus, the best evidence rule does not apply. D. The court should overrule the objection because the central issue in the case concerns the terms of the agreement, and the jury is entitled to hear defendant's interpretation of the terms.

A. The court should sustain the objection.

Plaintiff brings a civil action against defendant arising out of a hit-and-run accident. Defendant claims that although she drives the same kind of car that was involved in the accident, she had nothing to do with the accident. Defendant claims that X was the responsible party, and she subpoenas X to appear at trial. However, X refuses to testify (even when ordered to do so by the court), raising an invalid claim of privilege. Consequently, defendant wishes to call witness to testify that an hour after the accident he was introduced to X for the first time, and that X told witness that he (X) had just been involved in an accident and "took off before the cops arrived." Which of the following statements is most accurate? A. The evidence concerning X's statement to witness is hearsay but admissible as a declaration against interest. B. The evidence concerning X's statement is admissible as an admission. C. The evidence concerning X's statement is admissible as a present sense impression. D. The evidence concerning X's statement is inadmissible because X is not unavailable.

A. The evidence concerning X's statement to witness is hearsay but admissible as a declaration against interest.

Defendant is being prosecuted for assault and battery of victim. The attack took place at the corner of First and Main Streets. At trial, the prosecution calls witness, who witnessed the attack. After a few preliminary questions, the prosecutor asks witness where he was at on the evening the crime occurred. Witness says he cannot remember. The prosecutor then asks if witness was standing on the corner of First and Main Streets. Which of the following statements is most accurate? A. The question is unobjectionable. B. If defendant's attorney objects to the question as assuming a fact not in evidence, the court will sustain the objection. C. If defendant's attorney objects to the question as leading, the court will sustain the objection. D. If defendant's attorney objects on the basis that the question has already been asked and answered, the court will sustain the objection.

A. The question is unobjectionable.

Roberta Monk, a famous author, had a life insurance policy with Drummond Life Insurance Company. Her son, Peter, was beneficiary. Roberta disappeared from her residence in the city of Metropolis two years ago and has not been seen since. On the day that Roberta disappeared, Sky Airlines Flight 22 left Metropolis for Rio de Janeiro and vanished; the plane's passenger list included a Roberta Rector. Peter is now suing Drummond Life Insurance Company for the proceeds of his mother's policy. At trial, Peter offers to testify that his mother told him that she planned to write her next novel under the pen name of Roberta Rector. Peter's testimony is A. admissible as circumstantial evidence that Roberta Monk was on the plane. B. admissible as a party admission, because Roberta and Peter Monk are in privity with each other. C. inadmissible, because Roberta Monk has not been missing more than seven years. D. inadmissible, because it is hearsay not within any exception.

A. admissible as circumstantial evidence that Roberta Monk was on the plane.

In an arson prosecution the government seeks to rebut Defendant's alibi that he was in a jail in another state at the time of the fire. The government calls Witness to testify that he diligently searched through all the records of the jail and found no record of Defendant having been incarcerated there during the time Defendant specified. The testimony of Witness is A. admissible as evidence of absence of an entry from a public record. B. admissible as a summary of voluminous documents. C. inadmissible, because it is hearsay not within any exception. D. inadmissible, because the records themselves must be produced.

A. admissible as evidence of absence of an entry from a public record.

Park sued Dunlevy for copyright infringement for using in Dunlevy's book some slightly disguised house plans on which Park held the copyright. Park is prepared to testify that he heard Dunlevy's executive assistant for copyright matters say that Dunlevy had obtained an advance copy of the plans from Park's office manager. Park's testimony is A. admissible as reporting a statement of an employee of a party opponent. B. admissible as a statement of a co-conspirator. C. inadmissible, because it is hearsay not within any exception. D. inadmissible, because there is no showing that the assistant was authorized to speak for Dunlevy.

A. admissible as reporting a statement of an employee of a party opponent.

Dexter was tried for the homicide of a girl whose strangled body was found beside a remote logging road with her hands taped together. After Dexter offered evidence of alibi, the state calls Wilma to testify that Dexter had taped her hands and tried to strangle her in the same location two days before the homicide but that she escaped. The evidence is A. admissible, as tending to show Dexter is the killer. B. admissible, as tending to show Dexter's violent nature. C. inadmissible, because it is improper character evidence. D. inadmissible, because it is unfairly prejudicial.

A. admissible, as tending to show Dexter is the killer.

At defendant's murder trial, Defendant calls Witness as his first witness to testify that Defendant has a reputation in their community as a peaceable and truthful person. The prosecutor objects on the ground that Witness's testimony would constitute improper character evidence. The court should A. admit the testimony as to peaceableness, but exclude the testimony as to truthfulness. B. admit the testimony as to truthfulness, but exclude the testimony as to peaceableness. C. admit the testimony as to both character traits. D. exclude the testimony as to both character traits.

A. admit the testimony as to peaceableness, but exclude the testimony as to truthfulness.

Darby was prosecuted for sexually abusing his 13-year-old stepdaughter, Wendy. Wendy testified to Darby's conduct. On cross-examination, defense counsel asks Wendy, "Isn't it true that shortly before you complained that Darby abused you, he punished you for maliciously ruining some of his phonograph records?" The question is A. proper, because it relates to a possible motive for Wendy to accuse Darby falsely. B. proper, because Wendy's misconduct is relevant to her character for veracity. C. improper, because the incident had nothing to do with Wendy's truthfulness. D. improper, because it falls outside the scope of direct examination.

A. proper, because it relates to a possible motive for Wendy to accuse Darby falsely.

Phil is suing Dennis for injuries suffered in an automobile collision. At trial Phil's first witness, Wanda, testified that, although she did not see the accident, she heard her friend Frank say just before the crash, "Look at the crazy way old Dennis is driving!" Dennis offers evidence to impeach Frank by asking Wanda, "Isn't it true that Frank beat up Dennis just the day before the collision?" The question is A. proper, because it tends to show the possible bias of Frank against Dennis. B. proper, because it tends to show Frank's character. C. improper, because Frank has no opportunity to explain or deny. D. improper, because impeachment cannot properly be by specific instances.

A. proper, because it tends to show the possible bias of Frank against Dennis.

While walking down a city street, the plaintiff was seriously injured when a rotten limb fell off of a tree and hit him on the head. The tree was located on a vacant lot next to the defendant's house. The lot appeared to be part of the defendant's property. The plaintiff sued the defendant to recover damages for his injuries, alleging that the defendant was negligent with respect to the care of the tree. The defendant's defense was that he did not own the lot or the tree, and that both the lot and the tree were the property of the city. At trial, the plaintiff calls a witness to testify that shortly after the plaintiff was taken to the hospital, he observed the defendant cutting down the rotten limbs on a number of trees on the vacant lot. The witness's testimony is most likely: A. Admissible, to help prove that the defendant was negligent in not removing the rotten limbs sooner. B. Admissible, to help prove that the defendant owned the lot. C. Inadmissible, because subsequent repairs are encouraged for reasons of public safety. D. Inadmissible, because the evidence does not prove that the defendant owned the lot.

B. Admissible, to help prove that the defendant owned the lot.

D is on trial for murder. At trial, the prosecution offers against D a written statement prepared by X (a witness to the crime) at the request of the investigating police officer. X is now dead. X's statement includes a description of the perpetrator, and the characteristics match those of D. The court holds that the written statement is hearsay that does not fit within the regular exceptions to the hearsay rule, but that it satisfies the residual exception (FRE 807). D claims that the evidence should not be admitted because it would violate her confrontation rights under the Sixth Amendment. Which of the following statements is most likely correct? A. Because the evidence satisfies the requirements of the residual hearsay exception, which include considerations of reliability, X's statement is admissible despite the Confrontation Clause. B. Because X's statement is testimonial in nature, because D never had a chance to cross-examine X, and because X is unavailable, the court must exclude the statement. C. Because X's statement is not testimonial, the court may admit it despite the Confrontation Clause. D. Because the residual exception is not "firmly rooted," the court may not admit X's statement unless it finds the statement trustworthy after an independent inquiry.

B. Because X's statement is testimonial in nature, because D never had a chance to cross-examine X, and because X is unavailable, the court must exclude the statement.

A plaintiff sued a defendant in connection with the dissolution of a partnership they had formed to run a parcel delivery service. They had relied on a business attorney in establishing the business. After the business failed, the plaintiff and the defendant disagreed about their respective obligations. At trial, both have hired new counsel. The plaintiff calls the business attorney to testify to representations the defendant made in meetings she had with the plaintiff and the business attorney. The defendant objects to the business attorney's testimony, invoking the attorney-client privilege. Should the court uphold the defendant's privilege claim? A. No, because the business attorney's professional relationship with the plaintiff and the defendant has ended. B. No, because the plaintiff and the defendant consulted the business attorney jointly. C. Yes, because either the plaintiff or the defendant may block disclosure of statements made during such meetings. D. Yes, because either the plaintiff or the defendant may claim the privilege on behalf of the partnership.

B. No, because the plaintiff and the defendant consulted the business attorney jointly.

Roger robbed a bank with another man. When they were arrested, Roger made a deal with the prosecutor to become a cooperating witness and testify against his accomplice. At the accomplice's trial, Roger testified that he was only the getaway driver, and the accomplice entered the bank and shot the bank guard. On cross-examination, accomplice's defense counsel brought out that the prosecution had promised to drop the charges against Roger, put him in the witness protection program, and set him up with a new identity and employment. The defense lawyer suggested that Roger had made up the story against her client in order to please the government and get a good deal for himself. The prosecutor then called Roger's brother to testify that before he knew he was suspected of involvement in the robbery, Roger had confessed his participation in the robbery to him, supplying the same details about his participation and that of the accomplice he had testified to in court. The defendant's objection should be: A. Sustained, because it is hearsay. B. Overruled, because the statement is admissible as a prior consistent statement. C. Overruled, but the court should give a limiting instruction that the statement may only be used to bolster the credibility of the witness. D. Sustained, because it is improper character evidence.

B. Overruled, because the statement is admissible as a prior consistent statement.

Plaintiff sues defendant for negligence arising out of an automobile collision. Plaintiff calls witness, who testifies that she was standing at the intersection waiting to cross the street when defendant ran a red light and struck plaintiff's car broadside. On cross-examination, defendant establishes that witness was toward the back of a large crowd on the sidewalk before the crash, and had only a partial view of the street, that witness was looking in the direction of the bright sun, that witness has poor eyesight and was not wearing her glasses at the time, and that just after the crash, someone told witness the defendant had run the red light. Defendant moves to strike witness's testimony due to a lack of personal knowledge, and asks the court to instruct the jury to disregard witness's testimony. Which of the following statements is correct? A. The court should deny the motion if it concludes that witness had first-hand knowledge of the accident. B. The court should deny the motion if it finds that there is sufficient evidence to support a finding that witness had first-hand knowledge of the accident. C. The court should deny the motion because whether witness had first-hand knowledge is always a question for the jury. D. The court should deny the motion regardless of whether witness had first-hand knowledge because witness has already testify about the crash.

B. The court should deny the motion if it finds that there is sufficient evidence to support a finding that witness had first-hand knowledge of the accident.

Which of the following items of evidence is LEAST likely to be admitted without a supporting witness? A. In a libel action, a copy of a newspaper purporting to be published by Defendant Newspaper Publishing Company. B. In a case involving contaminated food, a can label purporting to identify the canner as Defendant Company. C. In a defamation case, a document purporting to be a memorandum from the Defendant Company president to "All Personnel," printed on Defendant's letterhead. D. In a case involving injury to a pedestrian, a pamphlet on stopping distances issued by the State Highway Department.

C. In a defamation case, a document purporting to be a memorandum from the Defendant Company president to "All Personnel," printed on Defendant's letterhead.

The plaintiff was driving her daughter to school when their car was struck broadside by a car driven by the defendant at an intersection controlled in all directions by stop signs. The plaintiff and her daughter were taken by ambulance to the hospital. In a personal injury action brought by the plaintiff and her daughter against the defendant, pretrial discovery revealed that both cars were in perfect mechanical condition just before the accident, and the defendant was on his way home from work at the time of the accident, but had stopped off at a bar before he reached the intersection at which he struck the plaintiff's car. There is no witness available to testify as to how much the defendant had to drink at the bar that day. At trial, the plaintiff calls a co-worker of the defendant, who testifies over objection that the defendant has a reputation as a hard drinker who tolerates alcohol well, as witnessed by the co-worker at numerous events. Was it an error for the trial court to admit this testimony? A. Yes, because in a civil matter, evidence of a party's character may not be introduced until he has put his character at issue. B. Yes, because the plaintiff may not attempt to prove that the defendant acted in a particular way on one occasion in conformity with his reputation as to that behavior. C. No, because the co-worker had personal knowledge of the defendant's drinking habits from having observed him while drinking. D. No, because there exists no unbiased eyewitness who can testify as to how much the defendant actually drank at the bar before he had the accident with the plaintiff.

B. Yes, because the plaintiff may not attempt to prove that the defendant acted in a particular way on one occasion in conformity with his reputation as to that behavior.

Perez sued Dawson for damages arising out of an automobile collision. At trial, Perez called Minter, an eyewitness to the collision. Perez expected Minter to testify that she had observed Dawson's automobile for five seconds prior to the collision and estimated Dawson's speed at the time of the collision to have been 50 miles per hour. Instead, Minter testified that she estimated Dawson's speed to have been 25 miles per hour. Without finally excusing Minter as a witness, Perez then called Wallingford, a police officer, to testify that Minter had told him during his investigation at the accident scene that Dawson "was doing at least 50." Wallingford's testimony is A. admissible as a present sense impression. B. admissible to impeach Minter. C. inadmissible, because Perez may not impeach his own witness. D. inadmissible, because it is hearsay not within any exception.

B. admissible to impeach Minter.

Defendant is on trial for nighttime breaking and entering of a warehouse. The warehouse owner had set up a camera to take infrared pictures of any intruders. After an expert establishes the reliability of infrared photography, the prosecutor offers the authenticated infrared picture of the intruder to show the similarities to Defendant. The pictures are A. admissible, provided an expert witness points out to the jury the similarities between the person in the photograph and Defendant. B. admissible, allowing the jury to compare the person in the photograph and Defendant. C. inadmissible, because there was no eyewitness to the scene available to authenticate the photograph. D. inadmissible, because infrared photography deprives a defendant of the right to confront witnesses.

B. admissible, allowing the jury to compare the person in the photograph and Defendant.

Defendant is on trial for robbing a bank in State A. She testified that she was in State B at the time of the robbery. Defendant calls her friend, Witness, to testify that two days before the robbery Defendant told him that she was going to spend the next three days in State B. Witness's testimony is A. admissible, because the statement falls within the present sense impression exception of the hearsay rule. B. admissible, because a statement of plans falls within the hearsay exception for then-existing state of mind. C. inadmissible, because it is offered to establish an alibi by Defendant's own statement. D. inadmissible, because it is hearsay not within any exception.

B. admissible, because a statement of plans falls within the hearsay exception for then-existing state of mind.

Deetz was prosecuted for homicide. He testified that he shot in self-defense. In rebuttal, Officer Watts testified that he came to the scene in response to a telephone call from Deetz. Watts offers to testify that he asked, "What is the problem here, sir?" and Deetz replied, "I was cleaning my gun and it went off accidentally." The offered testimony is A. admissible, as an excited utterance. B. admissible, to impeach Deetz and as evidence that he did not act in self-defense. C. inadmissible, because of Deetz's privilege against self-incrimination. D. inadmissible, because it tends to exculpate without corroboration.

B. admissible, to impeach Deetz and as evidence that he did not act in self-defense.

Defendant is charged with murder in connection with a carjacking incident during which Defendant allegedly shot Victim while attempting to steal Victim's car. The prosecutor calls Victim's four-year-old son, whose face was horribly disfigured by the same bullet, to testify that Defendant shot his father and him. The son's testimony should be A. admitted, provided the prosecutor first provides evidence that persuades the judge that the son is competent to testify despite his tender age. B. admitted, provided there is sufficient basis for believing that the son has personal knowledge and understands his obligation to testify truthfully. C. excluded, because it is insufficiently probative in view of the son's tender age. D. excluded, because it is more unfairly prejudicial than probative.

B. admitted, provided there is sufficient basis for believing that the son has personal knowledge and understands his obligation to testify truthfully.

In a jurisdiction without a Dead Man's Statute, Parker's estate sued Davidson claiming that Davidson had borrowed from Parker $10,000, which had not been repaid as of Parker's death. Parker was run over by a truck. At the accident scene, while dying from massive injuries, Parker told Officer Smith to "make sure my estate collects the $20,000 I loaned to Davidson." Smith's testimony about Parker's statement is A. inadmissible, because it is more unfairly prejudicial than probative. B. inadmissible, because it is hearsay not within any exception. C. admissible as an excited utterance. D. admissible as a statement under belief of impending death.

B. inadmissible, because it is hearsay not within any exception.

Defendant is charged with assault and battery. Defendant claims misidentification. Defendant calls his pastor as a witness. The pastor testifies that he has known defendant for most of his life, and that in his opinion defendant is a peaceful and nonviolent person who is not capable of committing the type of act with which he is charged. The prosecutor has credible evidence that defendant once attacked and severely beat a former employer. The prosecutor: A. May not ask the pastor whether he has heard about the prior attack and may not introduce any other evidence that the attack occurred. B. May ask the pastor whether he has heard that defendant once attacked a former employer, and, if the pastor denies having heard this, may call another witness to testify to the prior attack. C. May ask the pastor whether he has heard that defendant once attacked a former employer, but may not introduce other evidence of the attack. D. May not ask the pastor about the former attack unless criminal charges were brought against defendant for that attack.

C. May ask the pastor whether he has heard that defendant once attacked a former employer, but may not introduce other evidence of the attack.

At the defendant's trial for grand theft auto and other offenses, the prosecution offers to introduce the testimony of a police officer. The officer will testify that he showed a photographic lineup containing the defendant's picture to a witness who had seen the defendant fleeing from the stolen vehicle at the conclusion of a high-speed chase, and the witness selected the defendant's picture. The witness has left the state and refuses to return. Should the court admit the evidence? A.Yes, because the witness is unavailable to testify. B. Yes, because it is a prior identification. C. No, because it is inadmissible hearsay. D. No, because the picture has not been properly authenticated. -Which answer would be correct if the witness were present in court to testify?

C. No, because it is inadmissible hearsay. -(B)

The defendant is charged with soliciting an act of prostitution. The prosecutor calls a police officer who testifies that while working undercover he drove into a neighborhood where prostitutes were suspected of working. He slowed down and a provocatively clad woman, the defendant, approached his car. He rolled down the window and said, "Hey there." The officer would testify that she said, "I'll have sex with you for two hundred dollars." Does the officer's testimony about what she said fit within the definition of hearsay? A. Yes, because the defendant's statements are offered for the truth of an assertion. B. No, because there was no assertion. C. No, because it was a legally operative fact. D. No, because the defendant was the declarant.

C. No, because it was a legally operative fact.

Following her murder conviction, a juror approaches defendant's attorney and complains about several things that occurred during deliberations. If defendant makes a motion for a new trial, and asks the court to hold a hearing at which the juror may testify, which of the following would the court permit the juror to describe at the hearing? A. Several of the jurors announced at the beginning of deliberations that they made up their minds right at the end of the prosecution's case-in-chief that the defendant was guilty, and that they paid little attention to the defendant's case. B. During a lunch break while the trial was going on, several of the jurors drank beer to the point of obvious intoxication. C. One of the jurors told the other jurors that a publisher had approached her about a book deal if the jury voted to convict. D. One of the jurors made a disparaging remark about the defendant's unkempt appearance.

C. One of the jurors told the other jurors that a publisher had approached her about a book deal if the jury voted to convict.

A and B are on trial for the crimes of conspiracy and bank robbery. The judge has made a preliminary finding that the evidence is sufficient to prove that A and B conspired to commit bank robbery. Of the following statements, which is most likely to be admitted to prove the guilt of both A and B? A. Prior to the robbery, A, hoping to impress his girlfriend, told her that "B and I plan to rob First National Bank tomorrow." B. One week after the robbery, A voluntarily appeared at the local police station and informed a police officer that "B and I robbed First National Bank last week." C. One week before the robbery, A attempted to recruit C into the conspiracy by telling C, "B and I plan to rob First National Bank and we need you to drive the getaway car." D. One year after the robbery, A reveals to a friend that "B and I robbed First National Bank last year and got away with it."

C. One week before the robbery, A attempted to recruit C into the conspiracy by telling C, "B and I plan to rob First National Bank and we need you to drive the getaway car."

Parker sues Dix for breach of a promise made in a letter allegedly written by Dix to Parker. Dix denies writing the letter. Which of the following would NOT be a sufficient basis for admitting the letter into evidence? A. Testimony by Parker that she is familiar with Dix's signature and recognizes it on the letter. B. Comparison by the trier of fact of the letter with an admitted signature of Dix. C. Opinion testimony of a non-expert witness based upon familiarity acquired in order to authenticate the signature. D. Evidence that the letter was written in response to one written by Parker to Dix.

C. Opinion testimony of a non-expert witness based upon familiarity acquired in order to authenticate the signature.

Plaintiff has sued Defendant over an auto accident that resulted in damage to the vehicles but not in personal injury. In Plaintiff's lawsuit against Defendant, Plaintiff would testify that as soon as he got out of his car, Defendant said to Plaintiff: "Would you take a check for $5,000 to forget all of this?" Defendant's objection to this testimony should be: A. Sustained, because the statement was made in the course of attempting to settle a claim. B. Sustained, because Defendant's offer to pay for Plaintiff's damages does not conclusively prove that Defendant believed himself to be at fault for the accident. C. Overruled, because it is admissible to show Defendant's consciousness of his own negligence. D. Sustained, because it is improper character evidence.

C. Overruled, because it is admissible to show Defendant's consciousness of his own negligence.

In a federal investigation of Defendant for tax fraud, the grand jury seeks to obtain a letter written January 15 by Defendant to her attorney in which she stated: "Please prepare a deed giving my ranch to University but, in order to get around the tax law, I want it backdated to December 15." The attorney refuses to produce the letter on the ground of privilege. Production of the letter should be A. Prohibited, because the statement is protected by the attorney-client privilege. B. Prohibited, because the statement is protected by the client's privilege against self-incrimination. C. Required, because the statement was in furtherance of crime or fraud. D. Required, because the attorney-client privilege belongs to the client and can be claimed only by her.

C. Required, because the statement was in furtherance of crime or fraud.

Plaintiff sues defendant in a negligence action arising out of an automobile accident. Plaintiff was the sole occupant of her car, and defendant was the driver of a car in which witness was riding. Both plaintiff and witness were injured in the accident, and both sued defendant for their injuries. Prior to trial, defendant and witness settled, and at trial, witness testifies for defendant that plaintiff's car crossed the centerline and crashed into defendant's car. On cross-examination of witness, plaintiff wishes to inquire about the settlement between defendant and witness. Defendant objects. How should the court rule? A. The court should exclude the evidence. B. The court should admit the evidence only to prove defendant's negligence. C. The court should admit the evidence only to impeach witness by showing bias. D. The court should admit the evidence both to prove the defendant's negligence and to impeach witness by showing bias.

C. The court should admit the evidence only to impeach witness by showing bias.

Plaintiff sues attorney for legal malpractice. Attorney formerly represented plaintiff in a breach of contract case. Plaintiff lost that case, and now claims it was because of the incompetent representation by attorney. Attorney claims she represented plaintiff competently, and that early in the litigation process, she advised plaintiff to take certain action to maximize her chances of prevailing, but the plaintiff refused to follow her advice. At trial, attorney wishes to testify about the advice she gave plaintiff and plaintiff's response. Plaintiff objects on grounds of attorney-client privilege. How should the court rule? A. The court should sustain the objection because plaintiff is the holder of the privilege, and plaintiff has manifested her intention to maintain the privilege. B. The court should sustain the objection because the attorney-client privilege continues as long as there is a client in existence. C. The court should overrule the objection because there is no privilege for communications relevant to an alleged breach of duty by the attorney or client arising from the relationship. D. The court should sustain the objection as to statements made by plaintiff to attorney, but overrule the objection as to the advice attorney gave to plaintiff.

C. The court should overrule the objection because there is no privilege for communications relevant to an alleged breach of duty by the attorney or client arising from the relationship.

Assume each of the opinions listed below is relevant to a case. Which of the following opinions is a court least likely to allow a lay witness to give? A. The house smelled of gas just before the explosion. B. The car that passed us as we were trying to cross in the crosswalk was going at least 50 mph. C. The damage to the basement was caused by the water that seeped into the basement. D. The driver appeared to be intoxicated when he stepped out of the car moments after the crash.

C. The damage to the basement was caused by the water that seeped into the basement.

Under the rule allowing exclusion of relevant evidence because its probative value is substantially outweighed by other considerations, which of the following is NOT to be considered? A. The jury may be confused about the appropriate application of the evidence to the issues of the case. B. The evidence is likely to arouse unfair prejudice on the part of the jury. C. The opponent is surprised by the evidence and not fairly prepared to meet it. D. The trial will be extended and made cumbersome by hearing evidence of relatively trivial consequence.

C. The opponent is surprised by the evidence and not fairly prepared to meet it.

Plaintiff sues defendant for defamation. To prove the defamatory utterance was made, plaintiff calls Wilma, who testifies that she was in a group of plaintiff's business associates when defendant told the group that plaintiff was a "famous liar." During his case-in-chief, defendant calls Walter, Wilma's husband, to testify that Wilma suffers from moderate hearing loss and often forgets to wear her hearing aids. Which of the following statements is correct? A. Walter's testimony is admissible only if Wilma is given an opportunity to explain or deny the substance of Walter's testimony. B. Walter's testimony is admissible to impeach Wilma's character for truthfulness. C. Walter's testimony is admissible to impeach the accuracy of Wilma's testimony. D. Walter's testimony is inadmissible because extrinsic evidence may not be used in this situation.

C. Walter's testimony is admissible to impeach the accuracy of Wilma's testimony.

Plaintiff, a law professor, brings a wrongful discharge action against defendant university, his former employer. Plaintiff claims he was fired without cause. Defendant claims it fired plaintiff because rather than teaching Evidence, plaintiff taught quantum mechanics. Plaintiff denies this. To prove that plaintiff was teaching quantum mechanics, defendant calls witness, a student in plaintiff's class, to testify about the contents of a certain class session. Despite efforts to refresh witness's recollection, she cannot recall what took place on that day. Defendant then asked witness if she took notes during the class on that day and whether the notes accurately reflected what took place in the classroom. Witness answers "yes" to both questions. Defendant then asks witness to read aloud a page of her notes. The notes state that defendant discussed Schrodinger's cat, a quantum mechanics thought experiment. Plaintiff makes a hearsay objection. Which of the following statements is most accurate? A. Witness's notes are not hearsay because they are not offered to prove the truth of plaintiff's statements about quantum mechanics. The notes may be read into the record. B. Witness's notes are hearsay and because witness had no legal duty to record plaintiff's words accurately, no hearsay exception applies. C. Witness's notes are hearsay but may be read aloud by the witness under the doctrine of past recollection recorded. D. Witness's notes are not hearsay because defendant is merely using them to refresh the witness's recollection. They may therefore be read into the record.

C. Witness's notes are hearsay but may be read aloud by the witness under the doctrine of past recollection recorded.

In a civil action against defendant for sexually molesting Victim, a child, plaintiff (Victim's parent) in her case-in-chief wishes to call a Witness to testify (truthfully) that defendant molested Witness three years ago (when Witness was age 14). Defendant was never arrested nor convicted of molesting Witness. Which of the following is most likely correct? A. Witness's testimony is inadmissible hearsay. B. Witness's testimony is inadmissible because character evidence may not be admitted in a civil case. C. Witness's testimony is admissible character evidence. D. Witness's testimony is admissible because exclusion would violate Victim's due process rights.

C. Witness's testimony is admissible character evidence.

In a product liability action, plaintiff sued defendant, a manufacturer of an implantable medical device designed to regulate pancreatic function. A surgeon implanted the device in plaintiff, and plaintiff later developed pancreatic cancer. Plaintiff claims the device was the cause of the cancer. To prove causation, plaintiff calls witness, a Ph.D. in biochemistry who works for a major university. If permitted, witness will testify that based on certain animal studies she conducted on defendant's device, she has concluded that the device can cause pancreatic cancer. Defendant objects to witness's proposed testimony on the ground that it is not valid science. Which of the following best states the principle the court should use in ruling on the admissibility of witness's testimony? A. Witness's testimony will only be admissible if the court finds that it is based on a theory or method that is generally accepted in the relevant scientific community. B. Witness's testimony will only be admissible if the court finds that it is based on a theory or method that is generally accepted in the relevant scientific community and that the application of the theory or method is relevant to the case. C. Witness's testimony will only be admissible if the court finds that it is based on a reliable scientific theory or method and that the application of the theory or method is relevant to the case. D. Witness's testimony will only be admissible if it is based on a theory or method that has been subjected to publication and peer-review.

C. Witness's testimony will only be admissible if the court finds that it is based on a reliable scientific theory or method and that the application of the theory or method is relevant to the case.

Defendant is on trial for the murder of his father. Defendant's defense is that he shot his father accidentally. The prosecutor calls Witness, a police officer, to testify that on two occasions in the year prior to this incident, he had been called to Defendant's home because of complaints of loud arguments between Defendant and his father, and had found it necessary to stop Defendant from beating his father. The evidence is A. inadmissible, because it is improper character evidence. B. inadmissible, because Witness lacks firsthand knowledge of who started the quarrels. C. admissible to show that Defendant killed his father intentionally. D. admissible to show that Defendant is a violent person.

C. admissible to show that Defendant killed his father intentionally.

Plaintiff is suing Doctor for medical malpractice occasioned by allegedly prescribing an incorrect medication, causing Plaintiff to undergo substantial hospitalization. When Doctor learned of the medication problem, she immediately offered to pay Plaintiff's hospital expenses. At trial, Plaintiff offers evidence of Doctor's offer to pay the costs of his hospitalization. The evidence of Doctor's offer is A. admissible as a non-hearsay statement of a party. B. admissible, although hearsay, as a statement against interest. C. inadmissible, because it is an offer to pay medical expenses. D. inadmissible, because it is an offer to compromise.

C. inadmissible, because it is an offer to pay medical expenses.

Paulsen Corporation sued Dorr for ten fuel oil deliveries not paid for. Dorr denied that the deliveries were made. At trial, Paulsen calls its office manager, Wicks, to testify that Paulsen employees always record each delivery in duplicate, give one copy to the customer, and place the other copy in Paulsen's files; that he (Wicks) is the custodian of those files; and that his examination of the files before coming to court revealed that the ten deliveries were made. Wicks's testimony that the invoices show ten deliveries is A. admissible, because it is based on regularly kept business records. B. admissible, because Wicks has first-hand knowledge of the contents of the records. C. inadmissible, because the records must be produced in order to prove their contents. D. inadmissible, because the records are self-serving.

C. inadmissible, because the records must be produced in order to prove their contents.

Plaintiff sued Defendant for injuries sustained in an automobile collision. During Plaintiff's hospital stay, Doctor, a staff physician, examined Plaintiff's X-rays and said to Plaintiff, "You have a fracture of two vertebrae, C4 and C5." Intern, who was accompanying Doctor on her rounds, immediately wrote the diagnosis on Plaintiff's hospital record. At trial, the hospital records custodian testifies that Plaintiff's hospital record was made and kept in the ordinary course of the hospital's business. The entry reporting Doctor's diagnosis is A. inadmissible, because no foundation has been laid for Doctor's competence as an expert. B. inadmissible, because Doctor's opinion is based upon data that are not in evidence. C. admissible as a statement of then existing physical condition. D. admissible as a record of regularly conducted business activity.

D. admissible as a record of regularly conducted business activity.

A horse breeder offered to sell a colt to his neighbor and they agreed on a purchase price. The horse breeder subsequently received a letter from the neighbor thanking him for the sale and summarizing their agreement. The letter contained the neighbor's alleged signature. When the horse breeder attempted to set up transfer of the colt, the neighbor denied that she agreed to purchase it. In a breach of contract action against the neighbor, the horse breeder offers into evidence the letter. The horse breeder testifies that he is familiar with the neighbor's handwriting and recognizes the signature on the letter as being hers. Assuming appropriate objection by the neighbor, who claims that she did not sign the letter, the trial court should: A. Exclude the letter for lack of foundation because lay opinion testimony regarding handwriting identification is not admissible. B. Exclude the letter unless its authenticity is established by a preponderance of the evidence. C. Admit the letter as authentic and instruct the jury accordingly. D. Admit the letter but instruct the jury that it is up to them to decide whether the letter is authentic.

D. Admit the letter but instruct the jury that it is up to them to decide whether the letter is authentic.

In a contract suit between P and D, D testified that he recalls having his first conversation with P on January 23. When asked how he remembers the date, he answers, "In the conversation, P referred to an obituary of a mutual friend that was published in that day's newspaper." P's counsel moves to strike the reference to the newspaper. The judge should A. Grant the motion on the ground that the original writing rule requires production of the newspaper itself. B. Grant the motion because the reference to the newspaper story does not fit within any established exception to the hearsay rule. C. Deny the motion on the ground that the court may take judicial notice of local newspapers and their contents. D. Deny the motion on the ground that a witness may refer to collateral documents without providing the documents themselves.

D. Deny the motion on the ground that a witness may refer to collateral documents without providing the documents themselves.

The victim collapsed at her desk while drinking her morning coffee. Her secretary came rushing to her aid. Gasping for breath, the victim said, "I don't think I have much time left. I want you to remember when they come looking for suspects that I believe my assistant would kill for my job." The victim soon lost consciousness. She regained consciousness briefly after arriving at the hospital, but the doctors would not allow her to speak to anyone, including the police. She again lapsed into a coma, and she remains in a vegetative state. It was determined that she was poisoned. The assistant is arrested and charged with attempted murder. At the assistant's trial, the prosecution wishes to call the victim's secretary to testify to the victim's statement about the assistant before the ambulance arrived. The court should find the statement: A. Admissible, because it is a dying declaration. B. Admissible, because it is a declaration of the victim's state of mind. C. Admissible, as a statement of physical condition. D. Inadmissible, because it is hearsay not within any exception.

D. Inadmissible, because it is hearsay not within any exception.

At the trial of the plaintiff's breach of contract action against the defendant, the plaintiff called her accountant as a witness to testify about the difference in gross sales, gross income, and net profit caused by the defendant's failure to supply the promised quantity of ice cream to the plaintiff's ice cream shop. When the plaintiff's attorney asked the accountant to state the gross income figures for the year prior to the formation of the contract between the plaintiff and the defendant, the accountant replied he could not remember the exact amount. The plaintiff's counsel then handed the accountant a copy of the federal tax return submitted by the plaintiff for that year, and asked him to read it. Counsel then asked, "Now that you have read the tax return, can you remember what the gross income of the plaintiff's ice cream shop was for the relevant period?" The accountant answered, "Yes." Counsel then asked, "What was the gross income for that period?" The defendant's counsel objects. How should the court rule? A. Sustained, because the plaintiff's counsel is seeking to elicit testimony based on inadmissible hearsay. B. Sustained, because the accountant's testimony is not the best evidence. C. Overruled, because the accountant's hearsay testimony is admissible as a past recollection recorded. D. Overruled, because the accountant's testimony is admissible evidence relating to the plaintiff's damages.

D. Overruled, because the accountant's testimony is admissible evidence relating to the plaintiff's damages.

The plaintiff was badly injured in an automobile accident and spent several weeks recovering after he was released from the hospital. He has sued the driver of the other car for damages, including pain and suffering. At trial, the plaintiff's wife would testify that for six weeks after he came home from the hospital he complained of pain in his back and right hip every morning. The defendant's objection to the testimony should be: A. Sustained, hearsay. B. Sustained, the wife has no personal knowledge of the plaintiff's pain. C. Overruled, statement by a party opponent. D. Overruled, statement of then-existing physical condition.

D. Overruled, statement of then-existing physical condition.

In a personal injury action by P against D following an automobile collision, P claims that she suffered $25,000 in damages. D wishes to testify that a week before trial, P approached her and said, "You were negligent, but I was distracted by something on the side of the road and was looking away at the time of the crash. I'll dismiss the action if you'll pay me $5,000." Which of the following statements is most accurate? A. P's statement about being distracted is admissible as a party admission. The rest is inadmissible. B. P's entire statement is admissible as a party admission. C. P's entire statement is admissible as a declaration against interest. D. P's entire statement is inadmissible.

D. P's entire statement is inadmissible.

Plaintiff sued Defendant under an age discrimination statute, alleging that Defendant refused to hire Plaintiff because she was over age 65. Defendant's defense was that he refused to employ Plaintiff because he reasonably believed that she would be unable to perform the job. Defendant seeks to testify that Employer, Plaintiff's former employer, advised him not to hire Plaintiff because she was unable to perform productively for more than four hours a day. The testimony of Defendant is A. inadmissible, because Defendant's opinion of Plaintiff's abilities is not based on personal knowledge. B. inadmissible, because Employer's statement is hearsay not within any exception. C. admissible as evidence that Plaintiff would be unable to work longer than four hours per day. D. admissible as evidence for Defendant's reason for refusing to hire Plaintiff.

D. admissible as evidence for Defendant's reason for refusing to hire Plaintiff.

In January 2010, the defendant proposed to his girlfriend. During the engagement, the defendant confided in her about various drug deals in which he was participating. The woman swore that she would never reveal any of his confidences. On January 1, 2011, the couple married. The defendant continued to share with his wife information concerning his illegal drug activity. The wife's only rule was that he could not participate in any illegal drug transactions in their home. On one occasion in 2011, the wife came home unexpectedly and saw the defendant completing a drug transaction in the living room. The defendant was not aware that his wife had observed the event. In 2012, the defendant was charged by a federal prosecutor with 57 counts of illegal drug sales that occurred between 2009 and 2012. The prosecutor wishes to call the defendant's wife as a witness for the government. Assuming the defendant's attorney makes appropriate objections, which of the following statements is correct regarding the testimony by the defendant's wife? A. She can testify about the defendant's 2011 statements if she desires. B. She must testify to the defendant's 2010 statements. C. The defendant can keep her from testifying about his 2010 statements. D. She can testify to the drug sale that she observed in 2011 if she desires.

D. She can testify to the drug sale that she observed in 2011 if she desires.

In a negligence action, plaintiff sues defendant for an automobile accident arising from a collision at an intersection controlled by traffic signals. Plaintiff was driving east on one street, and defendant was driving north on the cross-street. Both parties claim that the other ran the light at the intersection, leading to the collision. At trial, plaintiff wishes to use a photograph to show the positions of the cars about 15 minutes after the collision. The photograph was taken by a passerby, who gave it to plaintiff. The passerby is not called to testify. Plaintiff seeks to authenticate the photograph by testifying that it accurately depicts the scene as it appeared immediately after the collision. Defendant objects, claiming lack of authentication. How should the court rule? A. The court should sustain the objection because plaintiff has not called the photographer to testify. B. The court should sustain the objection because the cars might have been moved between the time of the collision and the time the photograph was taken (15 minutes later). C. The court should sustain the objection because the photograph is not the best evidence of what happened at the time of the accident. D. The court should overrule the objection.

D. The court should overrule the objection.

Defendant is on trial for bank robbery. To prove that witness, not defendant, committed the crime, defendant calls witness and asks if witness committed the crime. Witness denies committing the crime. Defendant then asks witness if she admitted during her grand jury testimony that she had robbed the bank. Witness denies making such a statement. To prove witness's guilt, defendant wishes to offer witness's grand jury testimony admitting responsibility. Which of the following is correct? A. The evidence is inadmissible hearsay. B. The evidence is admissible as a party admission of witness. C. The evidence is admissible only if defendant did not know that witness would deny making the statement to the grand jury. D. The evidence is admissible non-hearsay to prove that witness committed the crime.

D. The evidence is admissible non-hearsay to prove that witness committed the crime.

Plaintiff sued Defendant for personal injuries arising out of an automobile accident. Which of the following would be ERROR? A. The judge allows Defendant's attorney to ask Defendant questions on cross-examination that go well beyond the scope of direct examination by Plaintiff, who has been called as an adverse witness. B. The judge refused to allow Defendant's attorney to cross-examine Defendant by leading questions. C. The judge allows cross-examination about the credibility of a witness even though no question relating to credibility has been asked on direct examination. D. The judge, despite Defendant's request for exclusion of witnesses, allows Plaintiff's eyewitness to remain in the courtroom after testifying, even though the eyewitness is expected to be recalled for further cross-examination.

D. The judge, despite Defendant's request for exclusion of witnesses, allows Plaintiff's eyewitness to remain in the courtroom after testifying, even though the eyewitness is expected to be recalled for further cross-examination.

A defendant is on trial for bribing a government procurement officer by providing the officer with free vacation facilities. When the defendant was approached by an FBI investigator, the defendant stated that her invitation to the procurement officer to spend his vacation in the defendant's mountain cabin was a favor to a friend, unrelated to his government office. The defendant also said to the investigator that she would reveal some "hot" information on a large-scale fraud in exchange for the investigator's promise to "stop worrying about a little vacation." Is the investigator's testimony about the defendant's offer to give information admissible? A. No, because it is hearsay not within any exception. B. No, because the defendant made the offer in a negotiation for settlement of a criminal investigation. C. Yes, as a matter observed and reported by the investigator pursuant to a duty imposed by law. D. Yes, as a statement of an opposing party.

D. Yes, as a statement of an opposing party.

A balloonist sued the manufacturer of deflation panels for hot air balloons after one of the panels failed while his balloon was descending, causing the balloon to crash and the balloonist to suffer severe injuries. At trial, the balloonist calls as a witness a structural engineer, who testifies, common to industry practice, her opinion is based on several reports done by an independent laboratory on the burst strength and material composition of the deflation panel closures. The balloonist's attorney then asks the engineer whether, in her opinion, the closures caused the deflation panel to give way. The manufacturer objects. Should the court admit this testimony? A. No, because the engineer did not perform the laboratory tests herself. B. No, because the laboratory reports are hearsay not within an exception. C. Yes, but the balloonist must offer into evidence the reports to which the engineer referred, so that the manufacturer may cross-examine as to them. D. Yes, because structural engineers reasonably rely on such reports in the course of their profession.

D. Yes, because structural engineers reasonably rely on such reports in the course of their profession.

Passenger is suing Defendant for injuries suffered in the crash of a small airplane, alleging that Defendant had owned the plane and negligently failed to have it properly maintained. Defendant has asserted in defense that he never owned the plane or had any responsibility to maintain it. At trial, Passenger calls Witness to testify that Witness had sold to Defendant a liability insurance policy on the plane. The testimony of Witness is A. inadmissible, because the policy itself is required under the original document rule. B. inadmissible, because of the rule against proof of insurance where insurance is not itself at issue. C. admissible to show that Defendant had little motivation to invest money in maintenance of the airplane. D. admissible as some evidence of Defendant's ownership of or responsibility for the airplane.

D. admissible as some evidence of Defendant's ownership of or responsibility for the airplane.

Defendant is on trial for extorting $10,000 from Victim. An issue is the identification of the person who made a telephone call to Victim. Victim is prepared to testify that the caller had a distinctive accent like Defendant's, but that he cannot positively identify the voice as Defendant's. Victim recorded the call but has not brought the tape to court, although its existence is known to Defendant. Victim's testimony is A. inadmissible, because Victim cannot sufficiently identify the caller. B. inadmissible, because the tape recording of the conversation is the best evidence. C. admissible, because Defendant waived the "best evidence" rule by failing to subpoena the tape. D. admissible, because Victim's lack of certainty goes to the weight to be given Victim's testimony, not to its admissibility.

D. admissible, because Victim's lack of certainty goes to the weight to be given Victim's testimony, not to its admissibility.

At Darrow's trial for stealing an automobile, Darrow called a character witness, Goode, who testified that Darrow had an excellent reputation for honesty. In rebuttal, the prosecutor calls Wick to testify that he recently saw Darrow cheat on a college examination. This evidence should be A. admitted, because Darrow has "opened the door" to the prosecutor's proof of bad character evidence. B. admitted, because the cheating involves "dishonesty or false statement." C. excluded, because it has no probative value on any issue in the case. D. excluded, because Darrow's cheating can be inquired into only on cross-examination of Goode.

D. excluded, because Darrow's cheating can be inquired into only on cross-examination of Goode.

Parr sued Davis for damages for physical injuries allegedly caused by Davis's violation of the federal civil rights law. The incident occurred wholly within the state of Chippewa but the case was tried in federal court. The Chippewa state code says, "The common-law privileges are preserved intact in this state." At trial, Davis called Dr. Webb, Parr's physician, to testify to confidential statements made to him by Parr in furtherance of medical treatment for the injuries allegedly caused by Davis. Parr objects, claiming a physician-patient privilege. The court should apply A. state law and recognize the claim of privilege. B. federal law and recognize the claim of privilege. C. state law and reject the claim of privilege. D. federal law and reject the claim of privilege.

D. federal law and reject the claim of privilege.

Plaintiff's estate sued Defendant Stores claiming that Guard, one of Defendant's security personnel, wrongfully shot and killed Plaintiff when Plaintiff fled after being accused of shoplifting. Guard was convicted of manslaughter for killing Plaintiff. At his criminal trial Guard, who was no longer working for Defendant, testified that Defendant's security director had instructed him to stop shoplifters "at all costs." Because Guard's criminal conviction is on appeal, he refuses to testify at the civil trial. Plaintiff's estate then offers an authenticated transcript of Guard's criminal trial testimony concerning the instructions of Defendant's security director. The evidence is A. admissible, as a statement of an agent of a party-opponent. B. admissible, because the instruction from the security director is not hearsay. C. admissible, although hearsay, as former testimony D. inadmissible, because it is hearsay not within any exception.

D. inadmissible, because it is hearsay not within any exception.

In a personal injury action by P, an apartment tenant, against D, the landlord, P claims to have fallen on a dark staircase leading from her second story apartment to the first floor. There was only minimal lighting on the stairs at the time of the accident. D claims that given the design of the stairwell, it was not possible to better illuminate the area. P takes the stand to testify that a month after the accident, D installed better lighting on the stairs. This evidence is: a. inadmissible evidence of subsequent remedial measures. b. inadmissible evidence of a specific instance of conduct to prove D's character. c. admissible to prove the feasibility of better lighting. d. admissible to prove that D was negligent.

c. admissible to prove the feasibility of better lighting.

In a personal injury action by P against D arising out of an automobile accident, D testifies at trial that he was driving carefully at the time of the accident. On cross-examination, P's attorney asks D, "isn't it true that your liability insurance covers you for $1 million per accident?" Under the facts as just related, this question is: a. proper because negligence may be inferred from D's having extensive liability coverage. b. proper because it is relevant to D's ability to satisfy any judgment against him. c. improper because it is irrelevant and potentially prejudicial. d. improper because plaintiffs can never ask about insurance coverage.

c. improper because it is irrelevant and potentially prejudicial.

One morning Tess telephoned her next-door neighbor Wilma and asked if she could borrow Wilma's car. Tess explained that her car was being serviced. Tess told Wilma that she had a doctor's appointment that afternoon and would return the car immediately afterwards. Wilma agreed to let Tess use her car. As Tess was driving to her appointment, she collided with a car driven by P. As a result of the accident, P brought suit against Wilma and Tess to recover for her personal injuries. P asserted a claim against Wilma for negligent entrustment of an automobile and charged Tess with negligent operation of a motor vehicle. In her case-in-chief, P called Vernon to testify to three incidents in which Wilma loaned her car to careless drivers during the past six months. The trial judge should rule Vernon's testimony a. admissible, as circumstantial evidence that Wilma was negligent on this occasion. b. admissible, since Vernon had personal knowledge of Tess's poor driving record. c. admissible against Wilma, but not Tess. d. inadmissible.

d. inadmissible.


संबंधित स्टडी सेट्स

Managerial Accounting- Test 1 Ch 2

View Set

E20-007 Data Science Associate Exam

View Set

Ch. 10 - Complications of pregnancy.

View Set

1. Acid-Base Balance: introduction

View Set

MRS. GRIFFITH'S - #2 ECONOMY -TRADE BARRIERS: Tariff, Quota, and Embargo

View Set

XCEL Chapter 11 - Laws and Rules (new)

View Set